Logo Studenta

Suponiendo que 0

💡 1 Respuesta

User badge image

Estudiando Tudo

Suponiendo que 0 < c < 1 si m≥n para todo m,n Є N.

¿Cómo puedo demostrar que 0 < c ᵐ≤ c ⁿ < 1?

NOTA IMPORTANTE: Si entendemos, como es usual en la mayoría de textos y autores, que 0 ∈ N, lo que se pide demostrar es falso: si m=n=0, c ⁿ=1,

de modo que no es c ⁿ < 1.

Además el cuantificador universal "PARA TODO" (∀) debe emplearse con sumo cuidado y a ser posible, antes de lo que "cuantifica", no al final. De hecho, entendiendo de manera literal la redacción, se condiciona al número c (real, suponemos) a la desigualdad 0 SI se cumple algo FALSO: que m≥n para todo m,n Є N.

Por ejemplo, m=5, n=7 son naturales, pero no es m≥n.

Si no tenemos precisión en matemáticas, ¿dónde la vamos a tener?

PROPONGO ESTA CORRECCIÓN EN LA PREGUNTA ( y respondo a ella):

Supóngase que 0 < c < 1, (c real), y m,n Є N, tales que m≥n>0.

¿Cómo puedo demostrar que 0 < c ᵐ≤ c ⁿ < 1?

DEMOSTRACIÓN:

Dada la doble desigualdad 0 < c < 1, por ser c positivo, podemos multiplicar por c todos sus términos, y se deduce:

0 < c² < c < 1; pero ahora, omitiendo la c intermedia, hemos probado que:

0 < c² < 1; multiplicando otra vez por c (positivo):

0 < c³ < c < 1, en particular (olvidando c): 0; vemos por el mismo razonamiento que todas las potencias de c, de exponente n natural, n>0, estarán comprendidas entre 0 y 1.

Pero para demostrarlo más formalmente, empleamos el método de inducción:

Si k es natural, k>0, se verifica 0 < c ᵏ< 1 (*)

DEMOSTRACIÓN:

1) La afirmación 0 < c ᵏ< 1 es verdadera cuando k=1,2,3.

2) HIPÓTESIS DE INDUCCIÓN:

Supongamos que para cierto k natural, k>0, se verifica 0 < c ᵏ< 1.

Multiplicando miembro a miembro esta última desigualdad por c (positivo), resulta

0 < c ᵏ⁺¹ < c < 1, y "olvidando" la c intermedia, 0 < c ᵏ⁺¹ < 1, de modo que suponiendo (*) válida para cierto k natural, k>0, será válida para el siguiente, k+1; y como es válida para k=1, por inducción, queda probado que (*) se cumplirá para todo k natural, tal que k>0.

Ahora bien, la pregunta indica que m y n son números naturales tales que m ≥ n.

Por tanto, si k=m-n, será k natural; luego, k ≥ 0 (pues podría ser m=n, y así k=0). Por la propiedad (*) antes demostrada, será 0 < c ᵏ≤ 1, es decir, 0 < c ᵐ⁻ ⁿ≤ 1.

[Los signos ≤ , de desigualdad amplia, se deben a que podría ser k=0, y en ese caso se daría la igualdad de los dos últimos miembros, y no por tanto la segunda desigualdad estricta de (*) ].

Pero sabemos que siendo n natural, es 0 < c ⁿ (&) ;

evidente: pues si n=0, se tiene c ⁰ = 1, si n=1 es c¹=c , y c > 0 por hipótesis; mientras que si n>1 se verifica 0 < c ⁿ por ser c ⁿ producto de números positivos. Multiplicando entonces por c ⁿ (positivo) los dos miembros de

0 < c ᵐ⁻ ⁿ≤ 1, tenemos: 0 < c ᵐ≤ c ⁿ. Pero por (*), siendo n natural, n>0, es c ⁿ < 1.

Así que hemos demostrado que:

si 0 < c < 1, (c real), y m,n Є N, tales que m≥n>0,

se verifica 0 < c ᵐ≤ c ⁿ<1,

que era lo que pedía la pregunta (corregida en su enunciado como se ha indicado).

0
Dislike0

✏️ Responder

FlechasNegritoItálicoSubrayadaTachadoCitaCódigoLista numeradaLista con viñetasSuscritoSobreDisminuir la sangríaAumentar la sangríaColor de fuenteColor de fondoAlineaciónLimpiarInsertar el linkImagenFórmula

Para escribir su respuesta aquí, Ingresar o Crear una cuenta

User badge image

Otros materiales

Preguntas relacionadas

Question Icon

¿Alguien puede demostrar que 1 sea igual que 0?

Geopolítica, Regionalização e Integração

User badge image

Aprendiendo a Aprender

Question Icon

¿Cómo probarían lo siguiente: ? X mayor que 0 entonces x + 1/x mayor o igual que 2

Geopolítica, Regionalização e Integração

User badge image

Aprendiendo a Aprender

Question Icon

¿Es posible si F= 0 que la aceleración sea diferente de 0?

Geopolítica, Regionalização e Integração

User badge image

Todos los Apuntes

Question Icon

¿Cómo se resuelve este ejercicio? 4√x+1-2√x+1+2=0

Geopolítica, Regionalização e Integração

User badge image

Materiales y Apuntes